Moderate exercise lowers the risk of blockage of the arteries due to blood clots, since anything that lowers blood ch...

farnoushsalimian on October 29, 2019

Why D?

Hi could you please explain why it's D Thank you

Replies
Create a free account to read and take part in forum discussions.

Already have an account? log in

SamA on October 29, 2019

Hello @farnoushsalimian,

Conclusion: Moderate exercise reduces the risk of blocked arteries due to blood clots.

How does the author reach this conclusion?

Premise 1: Anything that lowers blood cholesterol lowers risk of hardened arteries, which reduces risk of blood clot blockages.
Premise 2: According to a recent study, moderate exercise lowers blood cholesterol levels. (This is the key. It connects Premise 1 to the conclusion.)

What are the other effects of something that lowers blood cholesterol? It also reduces the risk of blocked arteries due to blood clots. If moderate exercise truly reduces blood cholesterol, then the conclusion can be properly drawn. The argument is sound, but only if we assume that the information from the study is accurate. I don't see any other flaws in the argument, or any other assumptions that need to be made. If we are not sure that moderate exercise belongs in the group from Premise 1, then the conclusion cannot be drawn.

jingjingxiao11111@gmail.com on April 10, 2020

I picked C because after I diagrammed it out, it became
conclusion: moderate exercise --- lowers the risk of blockage of the arteries due to blood clots
premise 1: moderate exercise --- lowers blood cholesterol
premise 2: Anything that lowers blood cholesterol -- lowers the risk of hardening of arteries --- lowers the risk of arterial blockage due to blood clots
C) says Lowering blood cholesterol levels lowers the risk of blockage of the arteries. which ties premise 1 and 2 together. So I thought C) was correct as an assumption. I never considered D) to be correct because I thought we are always supposed to assume data are correct in the premise. Thank you. Sorry for the long question.

Remi on January 28, 2021

I am also confused because the Stimulus already says "if the data reported in a recent study are correct", so shouldn't D already be eliminated since it is already assumed?